You are on page 1of 11

Winter 2012 Math 255

Problem Set 10
Section 17.1:
1) Find the gradient vector field of f (x, y, z) = xcos(y/z)
Solution:
f f f
f = i + j + k
x y z
x xy
= cos(y/z)i sin(y/z)j + 2 sin(y/z)k
z z
2) Give a formula F = M (x, y)i+N (x, y)j for the vector
field in the plane that has the properties that F =
0 at (0, 0) and that at any other point (a, b), F is
tangent to the circle x2 +y 2 = a2 +b2 and points
in the
clockwise direction with magnitude |F| = a2 + b2 .
Solution:

F = y i xk
Note that
F(0, 0) = 0
p
|F| = x2 + y 2
p
|F(a, b)| = a2 + b 2

Section 17.2
3) Integrate f (x, y, z) = (x + y + z)/(x2 + y 2 + z 2 ) over
the path r(t) = ti + tj + tk, 0 < a t b.
Solution:

1
Winter 2012 Math 255

r0 (t) = h1, 1, 1i

|r0 (t) = 3
t+t+t 1
f (r(t)) = 2 2 2
=
t +t +t t
Z Z b1
f ds = 3 dt = ln(b/a)
C a t

4) Find the mass of a wire that lies along the curve


r(t) = (t2 1)j + 2tk, 0 t 1, if the density is
= (3/2)t
Solution:

r0 (t) = h0, 2t, 2i, and |r0 (t) = 2 1 + t2

Z 1
0
Z 1 p
M= |r (t)|dt = 3 t 1 + t2 dt = 2 2 1
0 0

5) Evaluate C xydx + (x + y)dy along the curve y = x2


R

from (1, 1) to (2, 4).


Solution:
Parametrization:
r(t) = ti + t2 j 1 t 2
x = t, dx = dt; y = t2 , dy = 2tdt.
Z Z 2
3
t + (t + t2 ) 2t dt

xydx + (x + y)dy =
C 1
Z 2
 3 69
3 + 2t2 dt =

=
1 4

2
Winter 2012 Math 255

6) Find the circulation and flux of the field


F = y i + xj,
around and across the closed semicircular path that
consists of the semicircular arch r1 (t) = (acost)i +
(asint)j, 0 t , followed by the line segment
r2 (t) = ti, a t a.
Solution:
I Z Z
P dx+Qdy = P dx+Qdy+ P dx+Qdy = circulation
C C1 C2

C1 : x = acost, dx = asint dt
y = asint, dy = acost dt.
C2 : x = t, dx = dt, y = 0, dy = 0.
P = y = asint. Q = x = acost.
Z Z a
circulation = [asint(asint) + acost(acost)]dt + tdt
0 a
Z  2 a
2 t
= a dt + = a2
0 2 a
I Z Z
Flux = P dy Qdx = P dy Qdx + P dy Qdx
C
ZC1 C2

= [asint(acost) acost(asint)]dt
0Z
a
+ 0dt
a
= 0
(7) Find the flow of the velocity field
F = 4xy i + 8y j + 2k,

3
Winter 2012 Math 255

along the curve r(t) = ti + t2 j + k, 0 t 2.


Solution:

Z
Flow = Circulation = P dx + Qdy + Rdz
ZC2
= [4t2 (t2 ) + 8(t2 )(2t)]dt
Z0 2
= 4t3 dt = 16
0

Section 17.3
8) Show that the line integral C (1 yex )dx + ex dy,
R

is path independent and evaluate the integral. C is


any path from (0, 1)to(1, 2).
Solution:
Z Z
(1 yex )dx + ex dy = P dx + Qdy
C C
P
y = Q x
x = e , hence the line integral is path
independent. Then
Z
(1 yex )dx + ex dy = f (1, 2) f (0, 1),
C

where fx = (1 yex ) and fy = ex


fy = ex = f (x, y) = yex + g(x)
= fx = yex + g 0 (x) = 1 ex
= g 0 (x) = 1. Then g(x) = x + C
= f (x, y) = yex + x + C

4
Winter 2012 Math 255

Then
Z
(1 yex )dx + ex dy = f (1, 2) f (0, 1) = 2e1 .
C

9) Show that the differential form


Z (1,2,3)
2xydx + (x2 z 2 )dy 2yzdz,
(0,0,0)

is exact. Then evaluate the integral


Solution:
P
y = Q P R Q R
x = 2x, z = x = 0, z = y = 2z, hence
the differential form is exact. Then
Z (1,2,3)
2xydx+(x2 z 2 )dy2yzdz = f (1, 2, 3)f (0, 0, 0),
(0,0,0)

where fx = 2xy, fy = x2 z 2 , and fz = 2yz.


fx = 2xy = f (x, y, z) = x2 y + g(y, z)
= fy = x2 + gy = x2 z 2
= gy = z 2 . Then, g(y, z) = yz 2 + h(z)
= f (x, y, z) = x2 y yz 2 + h(z)
Then, fz = 2yz + h0 (z) = 2yz
= h0 (z) = 0 = h(z) = C
= f (x, y, z) = x2 y yz 2 + C
Then,
Z (1,2,3)
2xydx+(x2 z 2 )dy2yzdz = f (1, 2, 3)f (0, 0, 0) = 16,
(0,0,0)

10) Find the work done by F = (x2 +y)i+(y 2 +x)j+zez k


over the following paths from (1, 0, 0) to (1, 0, 1).

5
Winter 2012 Math 255

(a) The line segment x = 1, y = 0, 0 z 1.


(b) The helix r(t) = (cost)i + (sint)j + (t/2)k, 0
t 2.
(c) The x-axis from (1, 0, 0) to (0, 0, 0) followed by
the parabola z = x2 , y = 0 from (0, 0, 0) to
(1, 0, 1).
Solution:
P Q P R Q R
y = x = 1, z = x = 0, z = y = 0, hence F
is conservative and the work done is independent of
the path. Thus,
Z Z
Work = FTds = f Tds = f (1, 0, 1)f (1, 0, 0),
C C
for all the paths in (a), (b), and (c).

2 x3
fx = x + y = f (x, y, z) = + xy + g(y, z)
3
= fy = x + gy = y 2 + x
2 y3
= gy = y . Then, g(y, z) = + h(z)
3
x3 y3
= f (x, y, z) = + xy + + h(z)
3 3
Then, fz = h0 (z) = zez
Z
= h(z) = zez dz = zez ez + C
x3 y3
= f (x, y, z) = + xy + + zez ez + C.
3 3
Then,
Z Z
Work = FTds = f Tds = f (1, 0, 1)f (1, 0, 0) = 1.
C C

6
Winter 2012 Math 255

11) Suppose that F = f is a conservative vector field


and Z (x,y,z)
g(x, y, z) = F dr.
(0,0,0)
Show that g = F.
Solution: Since F = f ,
Z (x,y,z)
g(x, y, z) = F dr = f (x, y, z) f (0, 0, 0).
(0,0,0)

Then
g = {f (x, y, z) = f (0, 0, 0)} = f = F
Thus, g = F.
Section 17.4
12) Evaluate the line integral below along the given pos-
itively oriented curve using Greens Theorem.
Z
xe2x dx + (x4 + 2x2 y2)dy,
C

where C is the boundary of the region between the


circles x2 + y 2 = 1 and x2 + y 2 = 4.

7
Winter 2012 Math 255

Solution:
Z Z
2x 4 2 2
xe dx + (x + 2x y )dy = P dx + Qdy
C C  
ZZ
Q P
= dA
R x y
ZZ
= (4x3 + 4xy 2 )dA
R
Z 2 Z 2
= (4r3 cos3 + 4r3 cossin2 )rdrd
0
Z 2 1Z 2
= 4 (cos3 + cossin2 )r4 drd
Z0 2 1 Z 2
= 4 cosd r4 dr
0 1
= 0

13) Use Greens Theorem to find the counterclockwise


circulation and outward flux of the field F = xy i+y 2 j
around and over the boundary of the region enclosed
by the curves y = x2 and y = x in the first quadrant.
Solution:

I ZZ  
Q P
circulation = P dx + Qdy = dA
C x y
Z ZR
= (0 x)dA
R
Z 1Z x
= xdydx
0 x2
1
=
12

8
Winter 2012 Math 255

ZZ  
P Q
Flux = + dA
x y
Z ZR
= (y + 2y)dA
ZR1 Z x
= 3 ydydx
0 x2
1
=
5
14) Use a Greens Theorem area formula to find the area
of the region enclosed
by the curve r(t) = t2 i +
((t3 /3) t)j, 3 t 3.
Solution:
3
x = t2 , dx = 2t dt; y = t3 t, dy = (t2 1) dt.
ZZ
Area = dA
IR
1
= xdy ydx
2
Z 3  3

1 t
= t2 (t2 1) ( t)(2t) dt
2 3 3
Z 3  4 
1 t
= + t2 dt
2 3 3

8 3
=
5
Section 17.5
15) Which of the fields are conservative and which are
not? Find the curl and divergence of each field.

9
Winter 2012 Math 255

(i) F = y i + xj
(ii) F = yz i + xz j + xy k
(iii) F = (ysinz)i + (xsinz)j + (xycosz)k
(iv) F = y i + (x + z)j y k
Solution:

Recall that = x i + y j + z k

P Q
(i) y = 1 6= x = 1. Thus, the field is not con-
servative.
Divergence: F = 0
Curl: F = 2k

(ii) P
y = z = Q P R
x ; z = y = x ;
Q
z =x= R
y ,. Thus,
the field is conservative.
Divergence: F = 0
Curl: F = 0. Since F is conservative

(iii) P
y = sinz = Q P R Q
x ; z = ycosz = x ; z = xcosz =
R
y , Thus, the field is conservative.
Divergence: F = xysinz
Curl: F = 0. Since F is conservative

(iv) P
y = 1 = Q P R Q
x ; z = 0 = x ; but z = 1 6==
R
y = 1, Thus, the field is not conservative.
Divergence: F = 0
Curl: F = 2i.

10
Winter 2012 Math 255

16) Show that


(a) any vector field of the form F = f (x)i + g(y)j +
h(z)k where f , g, h are differentiable functions,
is irrotational.
(b) any vector field of the form F = f (y, z)i+g(x, z)j+
h(x, y)k is incompressible.
Solution:
(i)

i j k


F = x y

z
f (x) g(y) h(z)
 

= h(z) g(y) i
y z
 

h(z) f (x) j
x z
 

+ g(y) f (x) k
x y
= 0
Hence any vector field F = f (x)i + g(y)j + h(z)k
is irrotational.
(ii)

F= f (y, z) + g(x, z) + h(x, y) = 0.
x y z
Hence any vector field of the form F = f (y, z)i +
g(x, z)j + h(x, y)k is incompressible.

11

You might also like